Computing a series sumHow to prove $sum_{n=0}^{infty} frac{n^2}{2^n} = 6$?Writing an infinite series as the...

Can my American children re-enter the USA by International flight with a passport card? Being that their passport book has expired

Do people who work at research institutes consider themselves "academics"?

Understanding Deutch's Algorithm

Can I say: "When was your train leaving?" if the train leaves in the future?

How to continually let my readers know what time it is in my story, in an organic way?

How to redirect stdout to a file, and stdout+stderr to another one?

Why is the Advance Variation considered strong vs the Caro-Kann but not vs the Scandinavian?

How to rename multiple files in a directory at the same time

Meaning of "legitimate" in Carl Jung's quote "Neurosis is always a substitute for legitimate suffering."

Break long word (not long text!) in longtable cell

Would life always name the light from their sun "white"

What is the effect of the Feeblemind spell on Ability Score Improvements?

Why are lawsuits between the President and Congress not automatically sent to the Supreme Court

Is a "local" version of 3-SAT NP-hard?

Why did the soldiers of the North disobey Jon?

Will the volt, ampere, ohm or other electrical units change on May 20th, 2019?

Is there any good reason to write "it is easy to see"?

Were any toxic metals used in the International Space Station?

Using chord iii in a chord progression (major key)

Is the seat-belt sign activation when a pilot goes to the lavatory standard procedure?

How does a permutation act on a string?

Why would company (decision makers) wait for someone to retire, rather than lay them off, when their role is no longer needed?

My bread in my bread maker rises and then falls down just after cooking starts

Why doesn't Iron Man's action affect this person in Endgame?



Computing a series sum


How to prove $sum_{n=0}^{infty} frac{n^2}{2^n} = 6$?Writing an infinite series as the sum of the seriesComputing the sum of an infinite seriescomputing the series $sum_{n=1}^infty frac{1}{n^2 2^n}$Sum of a series to an exact answerConfused computing sum of Fourier seriesClosed form of this series?Prove that the given series is convergent.Sum of reciprocals of the square roots of the first N Natural NumbersSum function for power seriesShowing the sum of a power series is less than P$x$













3












$begingroup$


$$sum_{k=0}^infty frac{k^2}{3^k}$$



I tried with that 2 method but I couldn't get the $n^2$ term.










share|cite|improve this question









New contributor



Erinç Emre Çelikten is a new contributor to this site. Take care in asking for clarification, commenting, and answering.
Check out our Code of Conduct.






$endgroup$








  • 3




    $begingroup$
    You'll need to take two derivatives here and also make use of the first derivative case since taking two derivatives gives you $k(k-1) = k^2 - k$.
    $endgroup$
    – Cameron Williams
    7 hours ago












  • $begingroup$
    math.stackexchange.com/questions/593996/…
    $endgroup$
    – lab bhattacharjee
    4 hours ago










  • $begingroup$
    Possible duplicate of How to prove $sum_{n=0}^{infty} frac{n^2}{2^n} = 6$?
    $endgroup$
    – Hans Lundmark
    40 mins ago
















3












$begingroup$


$$sum_{k=0}^infty frac{k^2}{3^k}$$



I tried with that 2 method but I couldn't get the $n^2$ term.










share|cite|improve this question









New contributor



Erinç Emre Çelikten is a new contributor to this site. Take care in asking for clarification, commenting, and answering.
Check out our Code of Conduct.






$endgroup$








  • 3




    $begingroup$
    You'll need to take two derivatives here and also make use of the first derivative case since taking two derivatives gives you $k(k-1) = k^2 - k$.
    $endgroup$
    – Cameron Williams
    7 hours ago












  • $begingroup$
    math.stackexchange.com/questions/593996/…
    $endgroup$
    – lab bhattacharjee
    4 hours ago










  • $begingroup$
    Possible duplicate of How to prove $sum_{n=0}^{infty} frac{n^2}{2^n} = 6$?
    $endgroup$
    – Hans Lundmark
    40 mins ago














3












3








3





$begingroup$


$$sum_{k=0}^infty frac{k^2}{3^k}$$



I tried with that 2 method but I couldn't get the $n^2$ term.










share|cite|improve this question









New contributor



Erinç Emre Çelikten is a new contributor to this site. Take care in asking for clarification, commenting, and answering.
Check out our Code of Conduct.






$endgroup$




$$sum_{k=0}^infty frac{k^2}{3^k}$$



I tried with that 2 method but I couldn't get the $n^2$ term.







sequences-and-series power-series






share|cite|improve this question









New contributor



Erinç Emre Çelikten is a new contributor to this site. Take care in asking for clarification, commenting, and answering.
Check out our Code of Conduct.










share|cite|improve this question









New contributor



Erinç Emre Çelikten is a new contributor to this site. Take care in asking for clarification, commenting, and answering.
Check out our Code of Conduct.








share|cite|improve this question




share|cite|improve this question








edited 7 hours ago









David G. Stork

12.5k41836




12.5k41836






New contributor



Erinç Emre Çelikten is a new contributor to this site. Take care in asking for clarification, commenting, and answering.
Check out our Code of Conduct.








asked 7 hours ago









Erinç Emre ÇeliktenErinç Emre Çelikten

182




182




New contributor



Erinç Emre Çelikten is a new contributor to this site. Take care in asking for clarification, commenting, and answering.
Check out our Code of Conduct.




New contributor




Erinç Emre Çelikten is a new contributor to this site. Take care in asking for clarification, commenting, and answering.
Check out our Code of Conduct.










  • 3




    $begingroup$
    You'll need to take two derivatives here and also make use of the first derivative case since taking two derivatives gives you $k(k-1) = k^2 - k$.
    $endgroup$
    – Cameron Williams
    7 hours ago












  • $begingroup$
    math.stackexchange.com/questions/593996/…
    $endgroup$
    – lab bhattacharjee
    4 hours ago










  • $begingroup$
    Possible duplicate of How to prove $sum_{n=0}^{infty} frac{n^2}{2^n} = 6$?
    $endgroup$
    – Hans Lundmark
    40 mins ago














  • 3




    $begingroup$
    You'll need to take two derivatives here and also make use of the first derivative case since taking two derivatives gives you $k(k-1) = k^2 - k$.
    $endgroup$
    – Cameron Williams
    7 hours ago












  • $begingroup$
    math.stackexchange.com/questions/593996/…
    $endgroup$
    – lab bhattacharjee
    4 hours ago










  • $begingroup$
    Possible duplicate of How to prove $sum_{n=0}^{infty} frac{n^2}{2^n} = 6$?
    $endgroup$
    – Hans Lundmark
    40 mins ago








3




3




$begingroup$
You'll need to take two derivatives here and also make use of the first derivative case since taking two derivatives gives you $k(k-1) = k^2 - k$.
$endgroup$
– Cameron Williams
7 hours ago






$begingroup$
You'll need to take two derivatives here and also make use of the first derivative case since taking two derivatives gives you $k(k-1) = k^2 - k$.
$endgroup$
– Cameron Williams
7 hours ago














$begingroup$
math.stackexchange.com/questions/593996/…
$endgroup$
– lab bhattacharjee
4 hours ago




$begingroup$
math.stackexchange.com/questions/593996/…
$endgroup$
– lab bhattacharjee
4 hours ago












$begingroup$
Possible duplicate of How to prove $sum_{n=0}^{infty} frac{n^2}{2^n} = 6$?
$endgroup$
– Hans Lundmark
40 mins ago




$begingroup$
Possible duplicate of How to prove $sum_{n=0}^{infty} frac{n^2}{2^n} = 6$?
$endgroup$
– Hans Lundmark
40 mins ago










3 Answers
3






active

oldest

votes


















5












$begingroup$

$$frac{1}{1-x}=sum_{k=0}^{infty}x^k$$
$$frac{1}{(1-x)^2}=sum_{k=1}^{infty}kx^{k-1}$$
Multiply by $x$
$$frac{x}{(1-x)^2}=sum_{k=1}^{infty}kx^{k}$$
$$left(frac{x}{(1-x)^2}right)'=sum_{k=1}^{infty}k^2x^{k-1}$$
Multiply by $x$
$$xleft(frac{x}{(1-x)^2}right)'=sum_{k=1}^{infty}k^2x^{k}$$
then let $x=frac{1}{3}$






share|cite|improve this answer











$endgroup$





















    1












    $begingroup$

    Hint try to show
    begin{eqnarray*}
    sum_{k=0}^{infty} k^2 x^k =frac{x(1+4x+x^2)}{(1-x)^3}.
    end{eqnarray*}






    share|cite|improve this answer









    $endgroup$





















      0












      $begingroup$

      There is another way to deal with this problem.



      Denote $S_n=sum_{k=0}^n frac{k^2}{3^k}$ then$frac{1}{3}S_n=sum_{k=1}^{n+1} frac{(k-1)^2}{3^k}$ so $frac{2}{3}S_n=sum_{k=1}^nfrac{2k-1}{3^k}-frac{n^2}{3^{n+1}}$.



      In this way we change the numerator from twice power of $k$ to the lower power. Using the same operation you can change the numerator to numbers without the appearance of $k$ then you can use the summation formula for geometric series.






      share|cite|improve this answer









      $endgroup$














        Your Answer








        StackExchange.ready(function() {
        var channelOptions = {
        tags: "".split(" "),
        id: "69"
        };
        initTagRenderer("".split(" "), "".split(" "), channelOptions);

        StackExchange.using("externalEditor", function() {
        // Have to fire editor after snippets, if snippets enabled
        if (StackExchange.settings.snippets.snippetsEnabled) {
        StackExchange.using("snippets", function() {
        createEditor();
        });
        }
        else {
        createEditor();
        }
        });

        function createEditor() {
        StackExchange.prepareEditor({
        heartbeatType: 'answer',
        autoActivateHeartbeat: false,
        convertImagesToLinks: true,
        noModals: true,
        showLowRepImageUploadWarning: true,
        reputationToPostImages: 10,
        bindNavPrevention: true,
        postfix: "",
        imageUploader: {
        brandingHtml: "Powered by u003ca class="icon-imgur-white" href="https://imgur.com/"u003eu003c/au003e",
        contentPolicyHtml: "User contributions licensed under u003ca href="https://creativecommons.org/licenses/by-sa/3.0/"u003ecc by-sa 3.0 with attribution requiredu003c/au003e u003ca href="https://stackoverflow.com/legal/content-policy"u003e(content policy)u003c/au003e",
        allowUrls: true
        },
        noCode: true, onDemand: true,
        discardSelector: ".discard-answer"
        ,immediatelyShowMarkdownHelp:true
        });


        }
        });






        Erinç Emre Çelikten is a new contributor. Be nice, and check out our Code of Conduct.










        draft saved

        draft discarded


















        StackExchange.ready(
        function () {
        StackExchange.openid.initPostLogin('.new-post-login', 'https%3a%2f%2fmath.stackexchange.com%2fquestions%2f3226310%2fcomputing-a-series-sum%23new-answer', 'question_page');
        }
        );

        Post as a guest















        Required, but never shown

























        3 Answers
        3






        active

        oldest

        votes








        3 Answers
        3






        active

        oldest

        votes









        active

        oldest

        votes






        active

        oldest

        votes









        5












        $begingroup$

        $$frac{1}{1-x}=sum_{k=0}^{infty}x^k$$
        $$frac{1}{(1-x)^2}=sum_{k=1}^{infty}kx^{k-1}$$
        Multiply by $x$
        $$frac{x}{(1-x)^2}=sum_{k=1}^{infty}kx^{k}$$
        $$left(frac{x}{(1-x)^2}right)'=sum_{k=1}^{infty}k^2x^{k-1}$$
        Multiply by $x$
        $$xleft(frac{x}{(1-x)^2}right)'=sum_{k=1}^{infty}k^2x^{k}$$
        then let $x=frac{1}{3}$






        share|cite|improve this answer











        $endgroup$


















          5












          $begingroup$

          $$frac{1}{1-x}=sum_{k=0}^{infty}x^k$$
          $$frac{1}{(1-x)^2}=sum_{k=1}^{infty}kx^{k-1}$$
          Multiply by $x$
          $$frac{x}{(1-x)^2}=sum_{k=1}^{infty}kx^{k}$$
          $$left(frac{x}{(1-x)^2}right)'=sum_{k=1}^{infty}k^2x^{k-1}$$
          Multiply by $x$
          $$xleft(frac{x}{(1-x)^2}right)'=sum_{k=1}^{infty}k^2x^{k}$$
          then let $x=frac{1}{3}$






          share|cite|improve this answer











          $endgroup$
















            5












            5








            5





            $begingroup$

            $$frac{1}{1-x}=sum_{k=0}^{infty}x^k$$
            $$frac{1}{(1-x)^2}=sum_{k=1}^{infty}kx^{k-1}$$
            Multiply by $x$
            $$frac{x}{(1-x)^2}=sum_{k=1}^{infty}kx^{k}$$
            $$left(frac{x}{(1-x)^2}right)'=sum_{k=1}^{infty}k^2x^{k-1}$$
            Multiply by $x$
            $$xleft(frac{x}{(1-x)^2}right)'=sum_{k=1}^{infty}k^2x^{k}$$
            then let $x=frac{1}{3}$






            share|cite|improve this answer











            $endgroup$



            $$frac{1}{1-x}=sum_{k=0}^{infty}x^k$$
            $$frac{1}{(1-x)^2}=sum_{k=1}^{infty}kx^{k-1}$$
            Multiply by $x$
            $$frac{x}{(1-x)^2}=sum_{k=1}^{infty}kx^{k}$$
            $$left(frac{x}{(1-x)^2}right)'=sum_{k=1}^{infty}k^2x^{k-1}$$
            Multiply by $x$
            $$xleft(frac{x}{(1-x)^2}right)'=sum_{k=1}^{infty}k^2x^{k}$$
            then let $x=frac{1}{3}$







            share|cite|improve this answer














            share|cite|improve this answer



            share|cite|improve this answer








            edited 7 hours ago









            clathratus

            5,6861441




            5,6861441










            answered 7 hours ago









            E.H.EE.H.E

            17.5k11969




            17.5k11969























                1












                $begingroup$

                Hint try to show
                begin{eqnarray*}
                sum_{k=0}^{infty} k^2 x^k =frac{x(1+4x+x^2)}{(1-x)^3}.
                end{eqnarray*}






                share|cite|improve this answer









                $endgroup$


















                  1












                  $begingroup$

                  Hint try to show
                  begin{eqnarray*}
                  sum_{k=0}^{infty} k^2 x^k =frac{x(1+4x+x^2)}{(1-x)^3}.
                  end{eqnarray*}






                  share|cite|improve this answer









                  $endgroup$
















                    1












                    1








                    1





                    $begingroup$

                    Hint try to show
                    begin{eqnarray*}
                    sum_{k=0}^{infty} k^2 x^k =frac{x(1+4x+x^2)}{(1-x)^3}.
                    end{eqnarray*}






                    share|cite|improve this answer









                    $endgroup$



                    Hint try to show
                    begin{eqnarray*}
                    sum_{k=0}^{infty} k^2 x^k =frac{x(1+4x+x^2)}{(1-x)^3}.
                    end{eqnarray*}







                    share|cite|improve this answer












                    share|cite|improve this answer



                    share|cite|improve this answer










                    answered 7 hours ago









                    Donald SplutterwitDonald Splutterwit

                    23.4k21448




                    23.4k21448























                        0












                        $begingroup$

                        There is another way to deal with this problem.



                        Denote $S_n=sum_{k=0}^n frac{k^2}{3^k}$ then$frac{1}{3}S_n=sum_{k=1}^{n+1} frac{(k-1)^2}{3^k}$ so $frac{2}{3}S_n=sum_{k=1}^nfrac{2k-1}{3^k}-frac{n^2}{3^{n+1}}$.



                        In this way we change the numerator from twice power of $k$ to the lower power. Using the same operation you can change the numerator to numbers without the appearance of $k$ then you can use the summation formula for geometric series.






                        share|cite|improve this answer









                        $endgroup$


















                          0












                          $begingroup$

                          There is another way to deal with this problem.



                          Denote $S_n=sum_{k=0}^n frac{k^2}{3^k}$ then$frac{1}{3}S_n=sum_{k=1}^{n+1} frac{(k-1)^2}{3^k}$ so $frac{2}{3}S_n=sum_{k=1}^nfrac{2k-1}{3^k}-frac{n^2}{3^{n+1}}$.



                          In this way we change the numerator from twice power of $k$ to the lower power. Using the same operation you can change the numerator to numbers without the appearance of $k$ then you can use the summation formula for geometric series.






                          share|cite|improve this answer









                          $endgroup$
















                            0












                            0








                            0





                            $begingroup$

                            There is another way to deal with this problem.



                            Denote $S_n=sum_{k=0}^n frac{k^2}{3^k}$ then$frac{1}{3}S_n=sum_{k=1}^{n+1} frac{(k-1)^2}{3^k}$ so $frac{2}{3}S_n=sum_{k=1}^nfrac{2k-1}{3^k}-frac{n^2}{3^{n+1}}$.



                            In this way we change the numerator from twice power of $k$ to the lower power. Using the same operation you can change the numerator to numbers without the appearance of $k$ then you can use the summation formula for geometric series.






                            share|cite|improve this answer









                            $endgroup$



                            There is another way to deal with this problem.



                            Denote $S_n=sum_{k=0}^n frac{k^2}{3^k}$ then$frac{1}{3}S_n=sum_{k=1}^{n+1} frac{(k-1)^2}{3^k}$ so $frac{2}{3}S_n=sum_{k=1}^nfrac{2k-1}{3^k}-frac{n^2}{3^{n+1}}$.



                            In this way we change the numerator from twice power of $k$ to the lower power. Using the same operation you can change the numerator to numbers without the appearance of $k$ then you can use the summation formula for geometric series.







                            share|cite|improve this answer












                            share|cite|improve this answer



                            share|cite|improve this answer










                            answered 7 hours ago









                            Feng ShaoFeng Shao

                            18010




                            18010






















                                Erinç Emre Çelikten is a new contributor. Be nice, and check out our Code of Conduct.










                                draft saved

                                draft discarded


















                                Erinç Emre Çelikten is a new contributor. Be nice, and check out our Code of Conduct.













                                Erinç Emre Çelikten is a new contributor. Be nice, and check out our Code of Conduct.












                                Erinç Emre Çelikten is a new contributor. Be nice, and check out our Code of Conduct.
















                                Thanks for contributing an answer to Mathematics Stack Exchange!


                                • Please be sure to answer the question. Provide details and share your research!

                                But avoid



                                • Asking for help, clarification, or responding to other answers.

                                • Making statements based on opinion; back them up with references or personal experience.


                                Use MathJax to format equations. MathJax reference.


                                To learn more, see our tips on writing great answers.




                                draft saved


                                draft discarded














                                StackExchange.ready(
                                function () {
                                StackExchange.openid.initPostLogin('.new-post-login', 'https%3a%2f%2fmath.stackexchange.com%2fquestions%2f3226310%2fcomputing-a-series-sum%23new-answer', 'question_page');
                                }
                                );

                                Post as a guest















                                Required, but never shown





















































                                Required, but never shown














                                Required, but never shown












                                Required, but never shown







                                Required, but never shown

































                                Required, but never shown














                                Required, but never shown












                                Required, but never shown







                                Required, but never shown







                                Popular posts from this blog

                                Taj Mahal Inhaltsverzeichnis Aufbau | Geschichte | 350-Jahr-Feier | Heutige Bedeutung | Siehe auch |...

                                Baia Sprie Cuprins Etimologie | Istorie | Demografie | Politică și administrație | Arii naturale...

                                Nicolae Petrescu-Găină Cuprins Biografie | Opera | In memoriam | Varia | Controverse, incertitudini...